A kettle holds
4
44 liters of magic potion. There are
11
1111 liters of potion.
About how many kettles can we fill with magic potion?
Choose 1 answer:
Choose 1 answer:

(Choice A)
A
Between
0
00 and
1
11 kettles

(Choice B)
B
Between
1
11 and
2
22 kettles

(Choice C, Checked)
C
Between
2
22 and
3
33 kettles
Which equation tells us exactly how many kettles we can fill?
Choose 1 answer:
Choose 1 answer:

(Choice A)
A
11
÷
4
=
4
11
11÷4=
11
4

11, divided by, 4, equals, start fraction, 4, divided by, 11, end fraction

(Choice B)
B
4
÷
11
=
4
11
4÷11=
11
4

4, divided by, 11, equals, start fraction, 4, divided by, 11, end fraction

(Choice C)
C
11
÷
4
=
11
4
11÷4=
4
11

Answers

Answer 1

The correct option regarding the number of kettles that can be filled with the potion is given as follows:

C. Between 2 and 3.

How to obtain the number of kettles?

The number of kettles is obtained applying the proportions in this problem, dividing the total number of litters by the number of liters of each kettle.

The parameters for this problem are given as follows:

Total of 11 liters.Capacity of 4 liters per kettle.

Hence the number of kettles is obtained as follows:

11/4 = 2.75.

Meaning that the correct option is given by option C.

Missing Information

A kettle holds 4 liters of magic potion. There are 11 liters of potion.

More can be learned about proportions at https://brainly.com/question/24372153

#SPJ1


Related Questions

The equation, 868 = 700(1 + 0.04t), represents the amount of money earned on a savings account. What does the value 868 represent?
A. The value 868 represents the amount of interest earned, which means $868 was added to the initial balance.
B. The value 868 represents the final balance in the account, which means the account balance started with $868.
C. The value 868 represents the final balance in the account, which means the account ended with $868.
D. The value 868 represents the initial balance in the account, which means the account balance ended with $868.

Answers

The number 868 represents the final balance of the acount, then we conclude that the correct option is C.

"The value 868 represents the final balance in the account, which means the account ended with $868."

What does the value 868 represent?

Weknow that the equation 868 = 700(1 + 0.04t) represents the amount of money earned on a savings acount.

We want to see what does 868 represent in this situation.

Where the form of the equation is:

money earned = function that depends on t.

So the value at the left, 868, just represents the final balance of the account, this is the amount of money that there is in the account at the end.

Thus, the correct option is C.

Learn more about equations at:

https://brainly.com/question/13729904

#SPJ1

What is the weight (in grams) of a liquid that exactly fills a 182. 8 milliliter container if the density of the liquid is 0. 135 grams over milliliter ? Round to the nearest hundredth when necessary and only enter numerical values, which can include a decimal point

Answers

Liquid with density 0.135 grams over milliliter and volume 182.8 milliliter has weight equal to 24.68 grams.

Let 'm' represents the weight in grams of a liquid

Volume 'V' of the container to fill a liquid is = 182.8milliliter

Density 'ρ'  of the given liquid is 0.135 grams over milliliter

Density =  mass / Volume

Substitute the values we get,

⇒ 0.135 = m / 182.8 ×

⇒ m = 0.135  grams over milliliter ×  182.8milliliter

⇒ m = 24.678 grams

⇒ m = 24.68 grams

Therefore, the weight of a liquid for the given density and the volume is equal to 24.68 grams.

learn more about density here

brainly.com/question/6107689

#SPJ4

PLEASE HELP ASAP

Find the value of r so that the line through (r,5) and (6,r) has a slope of 5/8

Answers

Answer: 70/13

Step-by-step explanation:

Slope = rise/run

(r-5)/(6-r) = 5/8

30-5r = 8r-40

70 = 13r

70/13 = r

All sides of a lawn are expanded by the same amount so that it’s dimension are now 100 feet by 60 feet. If x is the same amount of length added to each side, write a function of x giving the length of each of the original sides. Use these functions to make a function for the area of the original lawn.

Answers

The function of x giving the length of each of the original sides would be  [tex](x + 100) * (x + 60) = x^2 + 160x + 6000[/tex].

What is Function of x?

A function is a formula that expresses one variable in terms of another. The statement "y is a function of x" (denoted y = y(x)) states that y changes depending on the value of x.

The original length of each side of the lawn can be represented by the function x + 100, and the original width can be represented by the function x + 60.

To find the area of the original lawn, we can multiply the length and width functions.

So, the function for the area of the original lawn is:

         [tex](x + 100) * (x + 60) = x^2 + 160x + 6000[/tex]

This is a quadratic function in x, whose leading coefficient is 1, and the value of x is the same amount of length added to each side of the lawn.

Hence, The function of x giving the length of each of the original sides would be  [tex](x + 100) * (x + 60) = x^2 + 160x + 6000[/tex].

To know more about Function of x visit,

https://brainly.com/question/17043948

#SPJ1

Which even number is not a composite number?
A) 6
B) 4
C) 2
D) 8

Answers

ithe answer will be a
kills ksksmslsls
im positive the answer is A6

Select all the ratios equivalent to 6:8
a)1:4
b)25:24
c)12:16​

Answers

Answer:

c)12:16​

Step-by-step explanation:

We know

The ratio is 6:8; we times 2 and get the ratio of 12:16

So, the answer is C

(e) (i) Using a scale of 2cm to represent 5 units on each axes, show, by shading the unwanted regions, the set of points satisfying the three inequalities in parts (b), (c) and (d). [3] (ii) Calculate the maximum number of rabbits the farmer can buy. [2]​

Answers

Answer: (e) (i) To represent the inequalities graphically on a coordinate plane, you can use a scale of 2cm to represent 5 units on each axis. To shade the unwanted regions, you can start by drawing the coordinate plane and labeling the x and y axes. Then, you can graph the inequalities by plotting the lines and shading the regions that do not satisfy the inequalities.

(b) 2x + 3y ≤ 30

This inequality represents a line with the equation 2x + 3y = 30. You can graph this line by plotting a few points and then drawing a straight line through them. The points that are on or below the line satisfy the inequality.

(c) x - 2y ≥ -10

This inequality represents a line with the equation x - 2y = -10. You can graph this line by plotting a few points and then drawing a straight line through them. The points that are on or above the line satisfy the inequality.

(d) y ≤ -2x + 10

This inequality represents a line with the equation y = -2x + 10. You can graph this line by plotting a few points and then drawing a straight line through them. The points that are on or below the line satisfy the inequality.

The set of points that satisfy all three inequalities is the area that is not shaded in the graph.

(ii) To find the maximum number of rabbits the farmer can buy, we need to find the coordinates of the vertex of the feasible region.

The coordinates of the vertex of the feasible region is (x,y) the point where the line 2x+3y=30 and x-2y=-10 intersects.

We can substitute the x-2y=-10 into the 2x+3y=30 equation

2x+3y=30

2x+3y+2y=-10

2x+5y=-10

x=-5y+10

substituting this into one of the equation we have

2(-5y+10)+3y=30

-10y+20+3y=30

-7y=10

y= -10/7

substituting this into x = -5y+10 we have

x = -5(-10/7)+10 = 15

the maximum number of rabbits the farmer can buy is 15

Step-by-step explanation:

a cell phone company has three different production sites. five percent of the phones from site 1, 7% from site 2, and 9% from site 3 have been recalled due to unexpected shutdown issue. suppose that 60% of the phones are produced at site 1, 30% at site 2, and 10% at site 3. if a randomly selected cell phone has been recalled, what is the probability that it came from site 3 (write it up to second decimal place)?

Answers

So, the probability that a randomly selected cell phone that has been recalled came from site 3 is 2.5 or 25%.

We can use Bayes' theorem to calculate the probability that a phone that has been recalled came from site 3. Bayes' theorem states that:

P(A|B) = P(B|A) * P(A) / P(B)

where:

P(A|B) is the probability of event A occurring given that event B has occurred (in this case, the probability that a phone came from site 3 given that it has been recalled)

P(B|A) is the probability of event B occurring given that event A has occurred (in this case, the probability that a phone has been recalled given that it came from site 3)

P(A) is the prior probability of event A occurring (in this case, the probability that a phone came from site 3)

P(B) is the prior probability of event B occurring (in this case, the probability that a phone has been recalled)

So, in this case, we have:

P(site 3 | recalled) = P(recalled | site 3) * P(site 3) / P(recalled)

We know that:

P(recalled | site 3) = 9%,

P(site 3) = 10%

We also know that:

P(recalled) = P(recalled | site 1) * P(site 1) + P(recalled | site 2) * P(site 2) + P(recalled | site 3) * P(site 3)

= (5% * 60%) + (7% * 30%) + (9% * 10%) = 3% + 2.1% + 0.9% = 6%

So, we can now calculate the probability that a phone that has been recalled came from site 3 as:

P(site 3 | recalled) = (9% * 10%) / 6% = 0.15 / 0.06 = 2.5

So the probability that a randomly selected cell phone that has been recalled came from site 3 is 2.5 or 25%.

To learn more about Bayes' theorem

Visit; brainly.com/question/28525447

#SPJ4

Solve for kkk.

20-6+4k=2-2k20−6+4k=2−2k20, minus, 6, plus, 4, k, equals, 2, minus, 2, k

k =k=k, equals

Answers

The value of k is -2

Now, According to the question:

The given equation is:

20 - 6 + 4k = 2 - 2k

We have to solve for the value of k.

Now,

20 - 6 + 4k = 2 - 2k

Combine the like terms;

20 - 6 - 2 = -2k - 4k

Calculate the sum or difference:

20 - 8 = -6k

12 = -6k

Calculate the product or quotient:

k = -12/6

k = -2

Hence, The value of k is -2

Learn more about Equation at:

https://brainly.com/question/24379564

#SPJ4

Give f(x)=3-|2x-5| find f(0)+f(-4)

Answers

The value of the function f(0) + f(-4) is -12

How to determine the value of the function?

From the question, we have the following equation that can be used to solve the question

f(x) = 3 - |2x - 5|

Also, we have the function values to be

f(0) and f(-4)

When these values are calculated, we have

f(0) = 3 - |2(0) - 5| = -2

f(-4) = 3 - |2(-4) - 5| = -10

substitute the known values in the above equation, so, we have the following representation

f(0) + f(-4) = -2 - 10

Evaluate

f(0) + f(-4) = -12

Hence, the solution is -12

Read more about function at

https://brainly.com/question/28277110

#SPJ1

Please help I need the answers ASAP.

Answers

Question 1:

[tex]\frac{2}{3} n-16=-\frac{5}{6}n+2[/tex], solve for n.

[tex]\frac{2}{3} n-16=-\frac{5}{6}n+2[/tex]

=> [tex]\frac{2}{3} n=-\frac{5}{6}n+18[/tex]

=> [tex]\frac{2}{3} n+\frac{5}{6}n=18[/tex] =>[tex]\frac{4}{6} n+\frac{5}{6}n=18[/tex]

=> [tex]\frac{9}{6} n=18[/tex] => [tex]\frac{3}{2} n=18[/tex]

=> [tex]n=18(\frac{2}{3})[/tex]

=> [tex]n=6(2)[/tex]

Sol: n=12

Question 2:

[tex]5-\frac{1}{3}p=\frac{4}{9}p+12[/tex], solve for p.

[tex]5-\frac{1}{3}p=\frac{4}{9}p+12[/tex]

=>[tex]5=\frac{4}{9}p+\frac{1}{3}p+12[/tex]

=> [tex]5=\frac{4}{9}p+\frac{3}{9}p+12[/tex]

=>[tex]5=\frac{7}{9}p+12[/tex]

=> [tex]-7=\frac{7}{9}p[/tex]

=> [tex]-7(\frac{9}{7}) =p[/tex]

=> [tex]-(9)=p[/tex]

Sol: p=-9

Question 3:

The relationship between ∠2 and ∠7 is that they are alternate interior angles.

Question 4:

The relationship between ∠1 and ∠3 is that they are corresponding angles.

Question 5:

The relationship between ∠4 and ∠5 is that they are alternate exterior angles.

what is the smallest even consecutive number such that their sum is at most 90?


SOLVE PLEASE

Answers

To find the smallest even consecutive numbers whose sum is at most 90, we can start with the smallest even number, which is 2. Then, we can add 2 to it to get the next even number, which is 4. We can keep adding 2 to get the next even numbers, and keep track of their sum. We can stop as soon as the sum exceeds 90.

The smallest even consecutive numbers whose sum is at most 90 are 2, 4, 6, 8, 10, 12, 14, 16, 18, 20, 22, 24, 26, 28, 30, 32, 34, 36, 38, 40, 42, 44, 46, 48, 50, 52, 54, 56, 58, 60, 62, 64, 66, 68, 70, 72, 74, 76, 78, 80, 82, 84, 86, 88, 90

The first 20 even consecutive numbers sum 90.

Francine had six times the money Tommy had, but after Francine gave Tommy $33 to help him purchase a baseball cap, Tommy had twice as much as Francine. How much did they each start with?

Answers

For the relation of times of money , Francine and Tommy start with the amount of money equals to $18 and $3 respectively.

Let 'x' represents the amount of money with Francine .

And 'y' represents the amount of money with Tommy.

In the first condition:

x = 6y __(1)

To purchase baseball cap Francine gave $33 then the given relation is :

y - 33 = 2( x - 33 )

⇒ y - 33 = 2x - 66

⇒y = 2x -66 + 33

⇒ y = 2x -33 __(2)

Substitute the value of 'x' from (1) in (2) we get,

y = 2(6y) - 33

⇒y = 12y - 33

⇒ 12y - y = 33

⇒ 11y = 33

⇒ y = $3

⇒ x = 6(3)

⇒ x = $18

Therefore, the amount of money Francine and Tommy start with is equal to $18 and $3 respectively.

Learn more about money here

brainly.com/question/29452742

#SPJ4

find the iqr of 81, 82, 83, 83, 84, 84, 84, 85

Answers

The Interquartile range of the given set is 1.5.

What is Interquartile range ?

The variation between the third and first quartiles is defined by the interquartile range. The numbers that partition the entire series into four equal sections are known as quartiles. There are therefore 3 quartiles. The lower quartile is represented by the letter Q₁, the higher quartile by the letters Q₁, and the middle quartile by the letter Q₃.

Given set : 81, 82, 83, 83, 84, 84, 84, 85

Here, It is already in increasing order but the number of values is 8 which is even.

So, Q₁ part = 81, 82, 83, 83

Q₁ will be the median of 82 and 83.

So, Q₁ = (82+83)/2

          = 82.5

Similarly, Q₃ part =84, 84, 84, 85

So, Q₃ will be the median of 84 and 84.

hence, Q₃ = (84+84)/2

                 = 84

We know that, Interquartile range = Q₃ - Q₁

                                                          =  84 - 82.5

                                                          = 1.5

Learn more about Interquartile range from the given link:

https://brainly.com/question/1210750

#SPJ1

Which is faster? Please show work
A. The black car that travels 110 feet per second
B. The silver car that travels 60 miles per hour

Answers

Answer:

the answer is A

I hope I help you in this ans and tq.

7:21:35 in its simplest form​

Answers

The smallest equivalent fraction of the number is considered to be its simplest form.The answer is  7/3 : 21/5 = 5 : 9.

What is what in its most basic form?

Values should now be entire numbers.By removing the denominators, convert fractions to integers.

Since the denominators of our two fractions are different, we must identify the least common denominator and rewrite our fractions using the common denominator as necessary.

LCD(7/3, 21/5) = 15

Now we have:

7/3 : 21/5 = 35/15 : 63/15

Now that the denominators of our two fractions are similar, we may multiply them both by 15 to get rid of the denominators.

Then, we have:

7/3 : 21/5 = 35 : 63

With the highest common factor, attempt to further reduce the ratio (GCF).

GCF is 7 for 35 and 63.

Divide both terms by GCF 7, then:

35 ÷ 7 = 5

63 ÷ 7 = 9

By multiplying both components by the GCF = 7, it is possible to reduce the ratio 35: 63 to its simplest terms.

35 : 63 = 5 : 9

Therefore:

7/3 : 21/5 = 5 : 9

To learn more about simplest form​ refer

https://brainly.com/question/21527775

#SPJ1

Function A and Function B are linear functions

Answers

to get the slope of any straight line, we simply need two points off of it, let's use those two in the picture below for Function A.

[tex](\stackrel{x_1}{-8}~,~\stackrel{y_1}{-5})\qquad (\stackrel{x_2}{10}~,~\stackrel{y_2}{4}) \\\\\\ \stackrel{slope}{m}\implies \cfrac{\stackrel{\textit{\large rise}} {\stackrel{y_2}{4}-\stackrel{y1}{(-5)}}}{\underset{\textit{\large run}} {\underset{x_2}{10}-\underset{x_1}{(-8)}}} \implies \cfrac{4 +5}{10 +8} \implies \cfrac{ 9 }{ 18 } \implies \cfrac{1 }{ 2 }\impliedby \stackrel{\textit{slope of}}{A} \\\\[-0.35em] ~\dotfill[/tex]

[tex]y = \stackrel{\stackrel{m}{\downarrow }}{5}x-1\qquad \impliedby \begin{array}{|c|ll} \cline{1-1} slope-intercept~form\\ \cline{1-1} \\ y=\underset{y-intercept}{\stackrel{slope\qquad }{\stackrel{\downarrow }{m}x+\underset{\uparrow }{b}}} \\\\ \cline{1-1} \end{array}\qquad \stackrel{\textit{slope of}}{B} \\\\[-0.35em] ~\dotfill\\\\ ~\hfill {\Large \begin{array}{llll} \stackrel{A}{\cfrac{1}{2}} ~~ < ~~ \stackrel{B}{5} \end{array}}~\hfill[/tex]

Jason is closing on a $430,000 home. He made a 13% down payment and is borrowing the rest. What is the approximate range of costs that he might expect to pay at the closing?

Answers

Answer:

$8,600 - $31,100

Step-by-step explanation:

Hope this helps :)

Consider the function g(x)=6cos(7x+22).

How much must x increase by for 7x+22 to increase by 2π?

Answers

After the increment in the function the value of x is -2.25

In math the term function is denoted as a relation from a set of inputs to a set of possible outputs where each input is related to exactly one output.

Here we know that the function g(x)=6cos(7x+22), x increase by for 7x+22 to increase by 2π.

In order to calculate the value of x in the function, we have to equate the value of function with 2π.

Then we get,

=> 7x + 22 = 2π

When we solve the equation for x then we get,

=> 7x = 2π - 22

=> x = (2π - 22)/7

When we simplify this one then we get the value of x as,

=> x = 0.285π - 3.14

=> x = 0.89 - 3.14

=> x = -2.25

To know more about function here.

https://brainly.com/question/28193995

#SPJ4

Write the missing angle measure next to each
letter.
65°
a, b, c, d, e
The angle relationship I used to find Angle a is...
The angle relationship I used to find Angle b is...
The angle relationship I used to find Angle c is...
The angle relationship I used to find Angle d is...
The angle relationship I used to find Angle e is...


Answers

The missing angle measure for the following letters are:

a - 65°

b - 115°

c - 115°

d - 65°

e - 65°

The angle relationship I used to find Angle a is the Corresponding Angle (a = 65°)The angle relationship I used to find Angle b is Angles on a Straight line (b = 180 - 65)The angle relationship I used to find Angle c is the Corresponding Angle (b = c)The angle relationship I used to find Angle d is Angle on a straight line (d = 180 - 65) = 115°The angle relationship I used to find Angle e is. Corresponding angles. d = e and e = d.

What is a corresponding angle?

Corresponding angles are generated when two parallel lines are crossed by a transversal.

Opening and closing a lunchbox, completing a Rubik's cube, and endless parallel railway tracks are all examples of equivalent angles.

Learn more about Angle Measure:
https://brainly.com/question/26612039
#SPJ1

Beer and lemonade are mixed in a ratio of 3:2 to make handy. 5% of a beer i alcohol. What percentage of a handy i alcohol?

Answers

The percentage of alcohol in a handy is 3%. 3/5 * 5% = 3% , Since beer is 5% alcohol, then 3/5 of the handy is 5% alcohol.

The ratio of beer to lemonade in a handy is 3:2, which means that 3 parts of the handy is beer and 2 parts of the handy is lemonade. To express it in terms of fractions, 3/5 of the handy is beer and 2/5 of the handy is lemonade. Since beer is 5% alcohol, then 3/5 of the handy is 5% alcohol. Therefore, if you take the proportion of the beer in the handy and multiply it by the alcohol percentage of the beer, you will get the percentage of alcohol in the handy. In this case, 3/5 * 5% = 3%. This means that the percentage of alcohol in a handy is 3%. It's important to note that this assumes that the beer and lemonade are mixed together evenly and there's no spillage or evaporation. Also, it's important to consider the fact that the alcohol percentage of the beer can vary depending on the brand and type of beer.

Learn more about percentage here

https://brainly.com/question/16797504

#SPJ4

Ayúdenme a encontrar el valor de los ángulos la medida de A) es 48 grados y E) es 136 grados y con eso encuentra los valores de los demás

Answers

eeeeeAnswer:

Step-by-step explanation:

se que g=136 t=48 los demás no los se

Car = 6 cylinder compact Years driven = second and third Miles driven = 13,000 miles in year two and 12,000 in year three In dollars and cents, the total projected depreciation cost for these two years would be

Answers

In dollars and cents, the total projected cost of gas and oil for two years would be $ 756.00

First we will calculate per year how much is spent.

First year, we have :-

= 13000 * 0.028 = 364.00

Second year, we have:-

12000 * 0.028 = 336.00

The total cost in the two years is the sum of both:

= first year + second year

= 364.00 + 336.00 = 700.00

In dollars and cents, the total projected cost of gas and oil for two years would be $ 700.00.

Total cost

Total cost refers to the overall cost of production, which includes both fixed and variable components of the cost.

Learn more about total cost here :-

https://brainly.com/question/28652728

#SPJ4

complete question is :-

Compute the requested operating costs as indicated, based on the preceding table and the following information.

Choose the correct answer.

Car = 6 cylinder compact

Years driven = second and third

Miles driven = 13,000 miles in year two and 12,000 in year three

In dollars and cents, the total projected depreciation cost for these two years would be $?

2x/3x-5-x+1/3x+5=-4/9x^2-25

Answers

The solution to the expression in this problem is given as follows:

x = -1/3 + 1.7i and x = -1/3 - 1.7i.

How to solve the rational expression?

The rational expression for this problem is defined as follows:

[tex]\frac{2x}{3x - 5} - \frac{x + 1}{3x + 5} = -\frac{4}{9x^2 - 25}[/tex]

Applying the least common factor at the left side, we have that:

[tex]\frac{2x(3x + 5) - (x + 1)(3x - 5)}{9x² - 25} = -\frac{4}{9x^2 - 25}[/tex]

[tex]\frac{6x^2 + 10x - 3x^2 + 5x - 3x + 5}{9x^2 - 25} = -\frac{4}{9x^2 - 25}[/tex]

As the denominators are equal, the solution is obtained equaling the denominators, hence:

3x² + 2x + 5 = -4

3x² + 2x + 9 = 0.

Which is a quadratic function with coefficients given as follows:

a = 3, b = 2, c = 9.

Inserting these coefficients into a calculator, the solutions are given as follows:

x = -1/3 + 1.7i and x = -1/3 - 1.7i.

Missing Information

The rational expression is given by the image presented at the end of the answer.

More can be learned about rational expressions at https://brainly.com/question/1851758

#SPJ1

Please help me <3 Thanks :)

Answers


The Answer is: The slope is 7/4

Willow is driving on the freeway. She just passed mile marker 325. The fastest she will drive is 75 miles per hour, though, there is a bit of traffic that might slow her down a bit, but she doesn’t know exactly how much. The mile markers are increasing on her route.


The expression 325 + 2y represents what mile marker she’s at after 2 hours, depending on traffic. Which inequality describes the mile marker she would be at this point?

Answers

The inequality describes the mile marker she would be at this point is:

325 + 2y ≤ 475

Now, According to the question:

Mathematical expressions with inequalities are those in which the two sides are not equal. Contrary to equations, we compare two values in inequality. Less than (or less than or equal to), greater than (or greater than or equal to), or not equal to signs are used in place of the equal sign.

We have,

The fastest driving speed of Willow is 75 miles per hour,

There is a bit of traffic on her route that might slow her speed down,

So, the inequality which represents the speed of the willow is:

y ≤ 75

Hence, the inequality which represents the speed of the driving of willow is y ≤ 75.

Now, the expression 361 + 2y represents what mile marker he is at after 2 hours. We know that the fastest he will drive is 75, so we could replace y=75 in the last expression to know what mile marker he would be seeing at that point. This is:

325 + 2(75) => 325 + 150 = 475

Now, we know that the inequality would be:

325 + 2y ≤ 475

Learn more about Inequality at:

https://brainly.com/question/30231190

#SPJ4

Choose the inequality that matches the graph below.

Answers

Answer:

X ≤ -1

How do we know this?

Well, if x = the value of any number in the graph, we can see that it can be equal to negative one, or less than -1, meaning we use the symbol ≤.

How do I work out this?

Answers

Answer:

3

Step-by-step explanation:

Take the x-intercept (1,0) and y-intercept (0,-3) to find the slope/gradient:

[tex]m=\frac{y_2-y_1}{x_2-x_1}=\frac{0-(-3)}{1-0}=3[/tex]

A set of elementary school student heights are normally distributed with a mean of 105 centimeters and a

standard deviation of 10 centimeters. Faisal is an elementary school student with a height of 103. 1 centimeters.

What proportion of student heights are lower than Faisal's height?

You may round your answer to four decimal places

Answers

Answer:

  0.4247

Step-by-step explanation:

You want to know the proportion of heights less than 103.1 cm if the population is normal with a mean of 105 cm and a standard deviation of 10 cm.

Proportion

The proportion can be found using a suitable probability calculator. The one shown in the attachment tells us ...

  the proportion of heights less than 103.1 cm is about 0.4247.

__

Additional comment

The height of interest is less than the mean by approximately 0.2 times the standard deviation. This suggests the proportion will be close to, but less than, 0.5000.

<95141404393>

A new Lexus sells for $56,000. Use 2 depreciation models and desmos to answer the following questions.

Model A: The car depreciates 12% per year.

Model B. Use straight line depreciation - the car has a value of $0 in 12 years.

Use desmos and attach a screenshot to show the following.

1. Use the wrench tool to label your x an y-values, change your x and y-value to appropriate numbers. Remember to lock the viewpoint.

2. Write on your screenshot and label the point where the linear model and the exponential model are equal.

3. Attach a screenshot to this ssignment.

Justgive me the equations

Answers

The time in years for which the values are the same is given as follows:

7.25 years.

How to obtain the models?

A decaying exponential model is given as follows:

y = a(1 - r)^t.

For which:

a is the initial value.r is the decay rate, as a decimal.

The parameters for this problem are given as follows:

a = 56000, r = 0.12.

Hence the exponential model is given as follows:

y = 56000(0.88)^x.

The linear model is given as follows:

y = b - mx.

In which:

b is the initial value.m is the yearly depreciation rate.

The parameter values for this problem are given as follows:

b = 56000, m = 56000/12 = 4666.67.

Hence the linear model is:

y = 56000 - 4666.67.

The point of intersection of the two graphs is at a time of 7.25 years.

More can be learned about exponential functions at https://brainly.com/question/25537936

#SPJ1

Other Questions
A new company president is saidto uave causethe company to do180. Before the new president, the company was losing 10,000 per month. What is the company most likely doing under the new president? A group of students was surveyed in a middle school class. They were asked how many hours they work on math homework each week. The results from the survey were recorded. Number of Hours Total Number of Students 0 1 1 3 2 3 3 10 4 9 5 6 6 3 Determine the probability that a student studied for exactly 5 hours. Round to the nearest hundredth. 0.83 0.21 0.17 0.14 Complete the following statement: Oxygen is a reactant of _____(1)_____ while being the product for _____(2)_____. (4 points) a(1) photosynthesis, (2) fermentation b(1) cellular respiration, (2) photosynthesis c(1) fermentation, (2) anaerobic respiration d(1) aerobic respiration, (2) photosynthesisPlease will give brainly!!! Complete the explanation of how you could use the work backward problem-solving strategy to solve the equation x/5 3 = 2.The equation says that a number was divided by 5 and that 3 was then subtracted from the quotient, giving the result 2. So, working backward, first add (blank)to 2, giving (blank). Then multiply that result by (blank), giving x = (blank) TRUE/FALSE. Argument by example examines specific cases in order to identify an actual relationship or correlation between them. As a public school teacher, Molly has accrued 14 days of sick leave, which is a type of ________ compensation. Group of answer choices:A) direct financialB) indirect financialC) nonfinancialD) mandatory Reggie wants to send an audio file to his colleague. However, he is unable to do so as the files are heavy in format. His friend asked him to convert the [BLANK] files to [BLANK] files explain in your own words how, in personal exchanges, altruistic preferences may affect the allocation of coffee and gigabytes between ayanda and bongani. what happens when fishery populations are overharvested? how have some governments curbed this trend? The family will eat together at the table what is the peoper and common noun??? cart 1 of mass m moves with a velocity of v toward cart 2 which has a mass of 9m and is at rest. cart 1 collides elastically with cart 2. what is the velocity of both carts after the collision? History shows us that one of the following is NOT a characteristic of most successful sport managers. For ease of data entry into a university database, 1 denotes that the student is a freshman, 2 indicates a sophomore, 3 indicates a junior, and 4 indicates that the student is a senior. In this case, data are___a cither categorical or quantitative. neither categorical nor quantitative.quantitative categorical Martin's mom has a large box of toys. The box contains 80 different toys. She wants to divide the large box into smaller boxes. She wants each of the smaller boxes to have 10 toys. After she divided the toys, she put 28 more toys into one of the boxes. How many toys are in that box? 90 is 0.6 of what number question 9 please help i need sm help lol ahhhhhhhh to be safe during a lightning storm, it is best to be to be safe during a lightning storm, it is best to be inside a wooden building without any grounding. inside a metal car. on a metal observation tower. next to a tall tree in a forest. in the middle of a grassy meadow. What is 1+0 I have no idea what the answer is These are business organizers that receive special favors from congress. People that start up their own business.